An alternate proof that sqrt(2) is irrational.

Поділитися
Вставка
  • Опубліковано 17 жов 2024

КОМЕНТАРІ • 43

  • @IgorBunar
    @IgorBunar 22 години тому +8

    Consider the polynomial x^2-2.
    The roots of the polynomial are both the negative and positive square root of 2.
    Notice that the polynomial has integer coefficients. From the rational roots theorem, any such polynomial's rational roots must be the possible to express as a factor of the constant over a factor of the leading term. We can list all such fractions: -1, 1, -2, 2. The square root of 2, does not belong to this set of possible rational roots, yet, it is a root of the polynomial in question. From this, it's possible to conclude that square root of 2 must be a non-rational root.

    • @tedszy7100
      @tedszy7100  21 годину тому +1

      Yes, rational roots theorem is a nice way!

  • @guyhoghton399
    @guyhoghton399 День тому +6

    Very interesting alternative proof.
    For the lemma about divisibility of a square by 2, I like the conciseness of this proof:
    _n(n - 1) ≡ 0 (mod 2)_ for any pair of consecutive integers _(n - 1)_ and _n_
    ⇒ _n² - n ≡ 0 (mod 2)_
    ⇒ _n ≡ n² (mod 2)_
    ∴ _2|n² ⇒ 2|n ⇒ (2×2)|(n×n) ⇒ 4|n²_

  • @skmaths-help
    @skmaths-help 2 дні тому +2

    Very nice proof - another super neat proof is as follows:
    Consider any polynomial with integer coefficients of the form a1 + a2\sqrt{2} + a2\sqrt{2}^2 + ...
    These expressions can all be written in the form a + b\sqrt{2} for integer a, b.
    Now consider the geometric series (\sqrt{2} - 1), (\sqrt{2}-1)^2, ...
    We have a common ration 0 < r < 1 thus we can choose arbitrarily small numbers of the form a + b\sqrt{2}.
    If root 2 is rational -> \sqrt{2} = m/n for positive integers m, n.
    a + b\sqrt{2} = (ay + bx)/n which must be greater than or equal to 1/n since but clearly we can choose a + b\sqrt{2} < 1/n so we are done.
    Just a really nice proof - thought I'd share!

    • @tedszy7100
      @tedszy7100  День тому

      Thanks. I'll have to sit down with a nice hot cup of green tea (no coffee) and think about this.

    • @annaclarafenyo8185
      @annaclarafenyo8185 День тому

      This is equivalent to the continued fraction proof, except folding in the theorem that best-approximations are in the continued fraction.

  • @simonwillover4175
    @simonwillover4175 6 годин тому

    The last step of the proof skips a lot of steps, but writing it out formally would take too long, so I think you did a great job.

  • @tedszy7100
    @tedszy7100  2 дні тому

    Like, comment, SUBSCRIBE! Follow me on FB: facebook.com/profile.php?id=61559517069850

  • @p0gr
    @p0gr День тому +2

    you proved everything very meticulously, but in the last step you assumed cancellation must be going on. why couldnt the denominators be different primes for example?

    • @simonwillover4175
      @simonwillover4175 6 годин тому

      Let a/b and c/d be rational numbers, where a/b = c/d, and neither are 0. Now, swap the variables if c>a. Example: with 2/4 = 6/3, we swap both fractions, getting 6/3 = 2/4. We now know what a>=c. If b c/d, which causes a contradiciton. Therefore, b>=d.
      The only way for a/b = c/d, regardless of whether the numbers are prime or not, is for there to exist some integer e>=1
      such that a=ce and b=de, so a/b = (c/d)(e/e). "Cancelling" term is mentioned because a/b = (ce)/(de), and the common term e can obviously be cancelled on both sides. But that means that a and b must have a common factor of e.
      In the video, a = m, b = n, c = 2n-m, and d = m-n. If m and n share a common factor of e, then gcd(m,n) must be a multiple of e. gcd(m,n) = 1 though, so the only valid value of e is 1. However, that means m = 2n-m, which is not possible since m > 2n-m. This causes a contradiction, so m/n can not = (2n-m)(m-n).

    • @p0gr
      @p0gr 5 годин тому +1

      @@simonwillover4175 The only way ... why? Thats exactly my point. It is of course true, but the proof is missing.

    • @grekiki
      @grekiki 3 години тому

      @@simonwillover4175 "The only way for a/b = c/d, regardless of whether the numbers are prime or not, is for there to exist some integer e>=1
      such that a=ce and b=de" I would really apreciate a proof of that

  • @simonwillover4175
    @simonwillover4175 7 годин тому

    1:38 for a more complete version of (p|ab -> p|a or p|b):
    ((p|ab) implies ((p|a) or (p|b))) *if and only if* ((p is prime) or (gcd(p,a) = 1) or (gcd(p,b) = 1) or (gcd(a,b) > p))
    The statement above is true, because it can be broken down into the following cases:
    ((p|ab) and (p is prime)) implies ((p|a) or (p|b))
    ((p|ab) and (gcd(p,a) = 1)) if and only if ((not (p|a)) and (p|b))
    ((p|ab) and (gcd(p,b) = 1)) if and only if ((p|a) and (not (p|b)))
    ((p|ab) and (gcd(a,b) > p)) if and only if ((p|a) and (p|b))
    The above statements are written using formal logic.
    By the way, "implies" refers to the right arrow operator "->", and "if and only if" is the 2-way version of it, "".

  • @DoxxTheMathGeek
    @DoxxTheMathGeek День тому

    Ohhh I really love that proof!!! :3
    I think the one by Euclid is still my favorite one though. ^^

  • @existenceispain2074
    @existenceispain2074 4 години тому

    The easiest way to prove this kind of result is to use fundamental theorem of arithmetic and notice that you can extend the theorem to rational numbers.(it is not a difficult corollary of the original result). The fundamental theorem of arithmetic is not a difficult theorem to prove.

  • @Avighna
    @Avighna День тому

    One problem I've always had with the "classic" proof is that we aren't using any properties of m,n being coprime while constructing this proof. We could just as well have assumed that gcd(m,n) = anything and we woulld still get the same result with no contradiction.

    • @ProactiveYellow
      @ProactiveYellow 20 годин тому

      Assume that GCD(m,n)>1. Because m and n are both positive integers, we know that their common divisor must be a natural number. Because the natural numbers have a lower limit (1 or 0 depending on your definition), there must be a finite amount of common factors between m and n. Because of how the classic argument is constructed, the contradiction arises as a proof by infinite descent. Even assuming a large common divisor, we should be able to only extract finitely many common factors, but we can find a factor of 2 infinitely many times, which implies we can get arbitrarily small pairs of natural numbers m,n, descending "infinitely," contradicting the well ordered nature of the natural numbers.

    • @Avighna
      @Avighna 12 годин тому

      @@ProactiveYellow That makes sense actually, yeah. I never thought of it that way.

  • @richardv.2475
    @richardv.2475 День тому

    Let's say it's trivial that (1) each positive number has one unique positive square root and (2) each positive rational number has exactly one unique simplest form where the nominator and the denominator are relatively prime and (3) if m and n are relatively prime then (m - n) and n are also relatively prime.
    Then you're basically saying that sqrt(2) = (2 - sqrt(2)) / (sqrt(2) - 1), so if sqrt(2) = m/n then sqrt(2) = (2 - m/n)/(m/n - 1) = (2n - m) / (m - n) and this is a contradiction because (m - n) and n are relatively prime, so once this new fraction is simplified into its simplest form it is clearly won't be m/n. And in general, this is applicable for all n because sqrt(n) = (n - sqrt(n)) / (sqrt(n) - 1) so if sqrt(n) = p / q then sqrt(n) = (n - p/q) /(p/q - 1) = (nq - p) / (p - q) and the only way to avoid contradiction is having a square number and q = 1, because then the relatively prime argument falls apart.
    Probably my high school textbook had a variant of this, so probably I'm also overcomplicating it.

    • @tedszy7100
      @tedszy7100  День тому

      Very nice, I like very much this idea of sqrt(2) = (2 - sqrt(2)) / (sqrt(2) - 1) and then putting in m/n for sqrt(2) on the right-hand side!

  • @rudycummings4671
    @rudycummings4671 9 годин тому

    I lost you when said, let's add thus to both sides. It is not crystal clear which equation you are referring to.

  • @annaclarafenyo8185
    @annaclarafenyo8185 День тому +1

    This proof is exactly the same as the classical proof. The best proof is to note that if you square (m/n) and you get an integer, you must get a square, by unique factorization of m and n into primes. The only reason people give more complicated proofs is because unique factorization into primes is a complicated theorem, so they want to present a proof that sidesteps it.
    An essentially different proof would be, for instance, using the fact that sqrt(2) has a periodic continued fraction, while a rational number must have a terminating continued fraction.

  • @erikev
    @erikev 18 годин тому +1

    This is just the same classical proof done in an aroundabout way.

  • @MrViktorWahrb
    @MrViktorWahrb День тому

    Hello! Such a good video. I'm in the 11:th grade but interested in more difficult math than the ones we do now:)

    • @tedszy7100
      @tedszy7100  День тому

      Thank you. I encourage you to continue with your interest, because there's so much fascinating math stuff accessible to 11th graders which is never touched on in class.

    • @DoxxTheMathGeek
      @DoxxTheMathGeek День тому

      Same, I'm in ninth grade, we are doing basic linear equations right now in school, it is pretty boring to me. X3
      Most of my teachers (!!!) don't even know what irrational means for numbers and call me stupid when I show them. qwq

    • @MrViktorWahrb
      @MrViktorWahrb День тому

      @@DoxxTheMathGeek If you want harder equations try learning some quadratic formulas, like the abc formula. Pythagorean theorem is a good one to learn. If you don’t know them that is. Start with pythagorean theorem:) If you know that try trigonometry! Cosine, sine, tangent. Very important in highschool here in Sweden at least.

    • @DoxxTheMathGeek
      @DoxxTheMathGeek День тому

      @@MrViktorWahrb I'm doing fractional calculus, I already know them. X3
      Thanks though! :3

    • @rapid4188
      @rapid4188 День тому

      oh brother college math is a roller coaster. you’ll have fun